Difference between revisions of "2023 AMC 12A Problems/Problem 24"

m (Solution)
 
(10 intermediate revisions by 5 users not shown)
Line 16: Line 16:
 
Let <math>f(x,\ell)</math> be the number of sequences <math>A_1</math>, <math>A_2</math>, <math>\dots</math>, <math>A_\ell</math> such that each <math>A_i</math> is a subset of <math>\{1, 2,\dots,x\}</math>, and <math>A_i</math> is a subset of <math>A_{i+1}</math> for <math>i=1</math>, <math>2\dots</math>, <math>\ell-1</math>. Then <math>f(x,1)=2^x</math> and <math>f(0,\ell)=1</math>.
 
Let <math>f(x,\ell)</math> be the number of sequences <math>A_1</math>, <math>A_2</math>, <math>\dots</math>, <math>A_\ell</math> such that each <math>A_i</math> is a subset of <math>\{1, 2,\dots,x\}</math>, and <math>A_i</math> is a subset of <math>A_{i+1}</math> for <math>i=1</math>, <math>2\dots</math>, <math>\ell-1</math>. Then <math>f(x,1)=2^x</math> and <math>f(0,\ell)=1</math>.
  
If <math>\ell\ge2</math> and <math>x\ge1</math>, we need to get a recursive formula for <math>f(x,\ell)</math>: If <math>|A_1|=i</math>, then <math>A_1</math> has <math>\text C_x^i</math> possibilities, and the subsequence <math>\{A_i\}_{2\le i\le\ell}</math> has <math>f(x-i,\ell-1)</math> possibilities. Hence <math>f(x,\ell)=</math>
+
If <math>\ell\ge2</math> and <math>x\ge1</math>, we need to get a recursive formula for <math>f(x,\ell)</math>: If <math>|A_1|=i</math>, then <math>A_1</math> has <math>\text C_x^i</math> possibilities, and the subsequence <math>\{A_i\}_{2\le i\le\ell}</math> has <math>f(x-i,\ell-1)</math> possibilities. Hence
 
<cmath>f(x,\ell)=\sum_{i=0}^x\text C_x^if(x-i,\ell-1).</cmath>
 
<cmath>f(x,\ell)=\sum_{i=0}^x\text C_x^if(x-i,\ell-1).</cmath>
 
By applying this formula and only considering  modulo <math>10</math>, we get <math>f(1,2)=3</math>, <math>f(1,3)=4</math>, <math>f(1,4)=5</math>, <math>f(1,5)=6</math>, <math>f(1,6)=7</math>, <math>f(1,7)=8</math>, <math>f(1,8)=9</math>, <math>f(1,9)=0</math>, <math>f(1,10)=1</math>, <math>f(2,2)=9</math>, <math>f(2,3)=6</math>, <math>f(2,4)=5</math>, <math>f(2,5)=6</math>, <math>f(2,6)=9</math>, <math>f(2,7)=4</math>, <math>f(2,8)=1</math>, <math>f(2,9)=0</math>, <math>f(2,10)=1</math>, <math>f(3,2)=7</math>, <math>f(3,3)=4</math>, <math>f(3,4)=5</math>, <math>f(3,5)=6</math>, <math>f(3,6)=3</math>, <math>f(3,7)=2</math>, <math>f(3,8)=9</math>, <math>f(3,9)=0</math>, <math>f(3,10)=1</math>, <math>f(4,2)=1</math>, <math>f(4,3)=6</math>, <math>f(4,4)=5</math>, <math>f(4,5)=6</math>, <math>f(4,6)=1</math>, <math>f(4,7)=6</math>, <math>f(4,8)=1</math>, <math>f(4,9)=0</math>, <math>f(4,10)=1</math>, <math>f(5,2)=3</math>, <math>f(5,3)=4</math>, <math>f(5,4)=5</math>, <math>f(5,5)=6</math>, <math>f(5,6)=7</math>, <math>f(5,7)=8</math>, <math>f(5,8)=9</math>, <math>f(5,9)=0</math>, <math>f(5,10)=1</math>, <math>f(6,2)=9</math>, <math>f(6,3)=6</math>, <math>f(6,4)=5</math>, <math>f(6,5)=6</math>, <math>f(6,6)=9</math>, <math>f(6,7)=4</math>, <math>f(6,8)=1</math>, <math>f(6,9)=0</math>, <math>f(6,10)=1</math>, <math>f(7,2)=7</math>, <math>f(7,3)=4</math>, <math>f(7,4)=5</math>, <math>f(7,5)=6</math>, <math>f(7,6)=3</math>, <math>f(7,7)=2</math>, <math>f(7,8)=9</math>, <math>f(7,9)=0</math>, <math>f(7,10)=1</math>, <math>f(8,2)=1</math>, <math>f(8,3)=6</math>, <math>f(8,4)=5</math>, <math>f(8,5)=6</math>, <math>f(8,6)=1</math>, <math>f(8,7)=6</math>, <math>f(8,8)=1</math>, <math>f(8,9)=0</math>, <math>f(8,10)=1</math>, <math>f(9,2)=3</math>, <math>f(9,3)=4</math>, <math>f(9,4)=5</math>, <math>f(9,5)=6</math>, <math>f(9,6)=7</math>, <math>f(9,7)=8</math>, <math>f(9,8)=9</math>, <math>f(9,9)=0</math>, <math>f(9,10)=1</math>, <math>f(10,2)=9</math>, <math>f(10,3)=6</math>, <math>f(10,4)=5</math>, <math>f(10,5)=6</math>, <math>f(10,6)=9</math>, <math>f(10,7)=4</math>, <math>f(10,8)=1</math>, <math>f(10,9)=0</math>, <math>f(10,10)=1</math>.
 
By applying this formula and only considering  modulo <math>10</math>, we get <math>f(1,2)=3</math>, <math>f(1,3)=4</math>, <math>f(1,4)=5</math>, <math>f(1,5)=6</math>, <math>f(1,6)=7</math>, <math>f(1,7)=8</math>, <math>f(1,8)=9</math>, <math>f(1,9)=0</math>, <math>f(1,10)=1</math>, <math>f(2,2)=9</math>, <math>f(2,3)=6</math>, <math>f(2,4)=5</math>, <math>f(2,5)=6</math>, <math>f(2,6)=9</math>, <math>f(2,7)=4</math>, <math>f(2,8)=1</math>, <math>f(2,9)=0</math>, <math>f(2,10)=1</math>, <math>f(3,2)=7</math>, <math>f(3,3)=4</math>, <math>f(3,4)=5</math>, <math>f(3,5)=6</math>, <math>f(3,6)=3</math>, <math>f(3,7)=2</math>, <math>f(3,8)=9</math>, <math>f(3,9)=0</math>, <math>f(3,10)=1</math>, <math>f(4,2)=1</math>, <math>f(4,3)=6</math>, <math>f(4,4)=5</math>, <math>f(4,5)=6</math>, <math>f(4,6)=1</math>, <math>f(4,7)=6</math>, <math>f(4,8)=1</math>, <math>f(4,9)=0</math>, <math>f(4,10)=1</math>, <math>f(5,2)=3</math>, <math>f(5,3)=4</math>, <math>f(5,4)=5</math>, <math>f(5,5)=6</math>, <math>f(5,6)=7</math>, <math>f(5,7)=8</math>, <math>f(5,8)=9</math>, <math>f(5,9)=0</math>, <math>f(5,10)=1</math>, <math>f(6,2)=9</math>, <math>f(6,3)=6</math>, <math>f(6,4)=5</math>, <math>f(6,5)=6</math>, <math>f(6,6)=9</math>, <math>f(6,7)=4</math>, <math>f(6,8)=1</math>, <math>f(6,9)=0</math>, <math>f(6,10)=1</math>, <math>f(7,2)=7</math>, <math>f(7,3)=4</math>, <math>f(7,4)=5</math>, <math>f(7,5)=6</math>, <math>f(7,6)=3</math>, <math>f(7,7)=2</math>, <math>f(7,8)=9</math>, <math>f(7,9)=0</math>, <math>f(7,10)=1</math>, <math>f(8,2)=1</math>, <math>f(8,3)=6</math>, <math>f(8,4)=5</math>, <math>f(8,5)=6</math>, <math>f(8,6)=1</math>, <math>f(8,7)=6</math>, <math>f(8,8)=1</math>, <math>f(8,9)=0</math>, <math>f(8,10)=1</math>, <math>f(9,2)=3</math>, <math>f(9,3)=4</math>, <math>f(9,4)=5</math>, <math>f(9,5)=6</math>, <math>f(9,6)=7</math>, <math>f(9,7)=8</math>, <math>f(9,8)=9</math>, <math>f(9,9)=0</math>, <math>f(9,10)=1</math>, <math>f(10,2)=9</math>, <math>f(10,3)=6</math>, <math>f(10,4)=5</math>, <math>f(10,5)=6</math>, <math>f(10,6)=9</math>, <math>f(10,7)=4</math>, <math>f(10,8)=1</math>, <math>f(10,9)=0</math>, <math>f(10,10)=1</math>.
Line 22: Line 22:
 
Lastly, we get <math>K\equiv\textstyle\sum\limits_{i=1}^{10}f(10,i)\equiv\boxed{\textbf{(C) } 5}\pmod{10}</math>.
 
Lastly, we get <math>K\equiv\textstyle\sum\limits_{i=1}^{10}f(10,i)\equiv\boxed{\textbf{(C) } 5}\pmod{10}</math>.
 
~Quantum-Phantom
 
~Quantum-Phantom
 +
==Solution 3 (Cheese, but this time it actually works)==
 +
Seeing that all the answers are different modulus 5, and that 10 is divisible by 5, we cheese this problem.
  
==Solution 3 (Cheese, answer by coincidence, incorrect logic) ==
+
Let <math>A_1, A_2, \cdots, A_n</math> be one sequence satisfying the constraints of the problem. Let <math>b_1, b_2, \cdots, b_n, b_{n+1}</math> be the sequence of nonnegative integers such that <math>A_k</math> has <math>b_k</math> elements for all <math>1\leq{}k\leq{}n</math>, and <math>b_{n+1}=10</math>. Note that we can generate the number of valid sequences of <math>A</math> by first generating all sequences of <math>b</math> such that <math>b_i\leq{}b_{i+1}</math> for all <math>1\leq{}i<n</math>, then choosing the elements from <math>A_{k+1}</math> that we keep in <math>A_k</math>, given the sequence of <math>b</math> as the restraint for the number of elements. For each sequence <math>b_1, b_2, \cdots{}, b_{n+1}</math>, there are <math>\prod_{i=2}^{n+1}\binom{b_i}{b_{i-1}}</math> corresponding sequences for <math>A</math>. Now, consider two cases - either all terms in <math>b</math> are either 0, 5, or 10, or there is at least one term in <math>b</math> that is neither 0, 5 nor 10. In the second case, consider the last term in <math>b</math> that is not 10 or 5, say <math>b_m</math>. However, that implies <math>b_{m+1}=10</math>, and so the number of corresponding sequences of <math>A</math> is <math>\binom{10}{b_m}\cdot{}</math> something or <math>\binom{5}{b_m}\cdot</math> something, which is always a multiple of <math>5</math>. Therefore, we only need to consider sequences of <math>b</math> where each term is <math>0</math> <math>5</math> or <math>10</math>. If all terms in <math>b</math> are 0 or 10, then for each <math>1\leq{}n\leq{}10</math> there are <math>n+1</math> sequences of <math>b</math> (since there are <math>n+1</math> places to turn from <math>0</math> to <math>10</math>), for a total of <math>2+3+\cdots+11=65\equiv0</math> (mod 5). If there exists at least one term <math>b_k=5</math>, then we use stars and bars to count the number of sequences of <math>b</math>, and each sequence of <math>b</math> corresponds to <math>\binom{10}{5}</math> sequences of <math>A</math>. For each <math>n</math>, we must have at least one term of <math>5</math>. After that, there are <math>n-1</math> stars and <math>2</math> bars (separating <math>0</math> to <math>5</math> and <math>5</math> to <math>10</math>), so that is <math>\binom{n+1}{2}</math> sequences of <math>b</math>. So the sum is <math>\binom{2}{2}+\binom{3}{2}+\cdots+\binom{11}{2}=\binom{12}{3}\equiv0</math> (mod 5). Therefore, the answer is 0 mod 5, and it must be <math>\boxed{\textbf{(C) } 5}</math>
Since the question only wants mod 10 of the answer, we can cheese this problem.
 
Let <math>b_n</math> be the number of elements of the set <math>a_n</math>. Assume that <math>b_k\neq0</math> and <math>b_k\neq10</math> for any <math>k</math>. However, that means by symmetry, there will be <math>\binom{10}{k}</math> different sequences of <math>a</math> with the same sequence of <math>b</math>. Since <math>\binom{10}{k}</math> is 0 mod 10 for all <math>k</math> except for 0 and 10, we only consider sequences where each term is either the empty set or the entire set <math>{1,2,\cdots,10}</math>. If <math>n=1</math>, then there are <math>2</math> sets. If <math>n=2</math>, then there are <math>3</math> sets, and so on. So the answer is <math>2+3+\cdots{}+11=65==5</math> (mod 10).
 
 
 
*Note: Unfortunately, <math>\binom{10}{5}</math> is not congruent to 0 mod 10, so this solution has the correct answer by coincidence. Also <math>\binom{10}{2}</math> and <math>\binom{10}{8}</math> are not 0 mod 10. Also, the negation of "there does not exist a <math>k</math> so that <math>b_k</math> is 0 or 10" is NOT "for all <math>k</math>, <math>b_k</math> is 0 or 10."
 
  
 
==Solution==
 
==Solution==
Line 42: Line 40:
 
</cmath>
 
</cmath>
  
 +
Recalling or noticing that  <math>x^n \equiv x^{n
 +
\mod 4} \pmod {10}</math>, then,
 
Modulo 10, we have
 
Modulo 10, we have
 
<cmath>
 
<cmath>
Line 54: Line 54:
  
 
~Steven Chen (Professor Chen Education Palace, www.professorchenedu.com)
 
~Steven Chen (Professor Chen Education Palace, www.professorchenedu.com)
 +
 +
Slightly easier, observe that <math>11^{10} \equiv 1^{10 }\pmod {10}</math>, so, working <math>{\mod 10}</math>, we have
 +
 +
<cmath>
 +
\begin{align*}
 +
K & \equiv \sum_{m = 2}^{11} m^2 \\
 +
& \equiv \sum_{m = 1}^{10} m^2 \\
 +
& \equiv \frac{10 \cdot \left( 10 + 1 \right) \left( 2 \cdot 10 + 1 \right)}{6} \\
 +
& \equiv \frac{10 \cdot 11 \cdot 21}{6} \\
 +
& \equiv 5 \cdot 11 \cdot 7 \\
 +
& \equiv \boxed{ 5}
 +
\end{align*}
 +
</cmath>
 +
 +
~oinava
  
 
==Video Solution 1 by OmegaLearn==
 
==Video Solution 1 by OmegaLearn==
 
https://youtu.be/0LLQW0XCKsQ
 
https://youtu.be/0LLQW0XCKsQ
 +
 +
==Video Solution==
 +
 +
https://youtu.be/FpagLq1uzBI
 +
 +
~Steven Chen (Professor Chen Education Palace, www.professorchenedu.com)
  
  
 
==See also==
 
==See also==
 
{{AMC12 box|ab=A|year=2023|num-b=23|num-a=25}}
 
{{AMC12 box|ab=A|year=2023|num-b=23|num-a=25}}
 +
 +
[[Category:Intermediate Combinatorics Problems]]
 
{{MAA Notice}}
 
{{MAA Notice}}

Latest revision as of 09:08, 19 February 2024

Problem

Let $K$ be the number of sequences $A_1$, $A_2$, $\dots$, $A_n$ such that $n$ is a positive integer less than or equal to $10$, each $A_i$ is a subset of $\{1, 2, 3, \dots, 10\}$, and $A_{i-1}$ is a subset of $A_i$ for each $i$ between $2$ and $n$, inclusive. For example, $\{\}$, $\{5, 7\}$, $\{2, 5, 7\}$, $\{2, 5, 7\}$, $\{2, 5, 6, 7, 9\}$ is one such sequence, with $n = 5$.What is the remainder when $K$ is divided by $10$?

$\textbf{(A) } 1 \qquad \textbf{(B) } 3 \qquad \textbf{(C) } 5 \qquad \textbf{(D) } 7 \qquad \textbf{(E) } 9$

Solution 1

Consider any sequence with $n$ terms. Every 10 number has such choices: never appear, appear the first time in the first spot, appear the first time in the second spot… and appear the first time in the $n$th spot, which means every number has $(n+1)$ choices to show up in the sequence. Consequently, for each sequence with length $n$, there are $(n+1)^{10}$ possible ways.

Thus, the desired value is $\sum_{i=1}^{10}(i+1)^{10}\equiv \boxed{\textbf{(C) } 5}\pmod{10}$

~bluesoul

Solution 2

Let $f(x,\ell)$ be the number of sequences $A_1$, $A_2$, $\dots$, $A_\ell$ such that each $A_i$ is a subset of $\{1, 2,\dots,x\}$, and $A_i$ is a subset of $A_{i+1}$ for $i=1$, $2\dots$, $\ell-1$. Then $f(x,1)=2^x$ and $f(0,\ell)=1$.

If $\ell\ge2$ and $x\ge1$, we need to get a recursive formula for $f(x,\ell)$: If $|A_1|=i$, then $A_1$ has $\text C_x^i$ possibilities, and the subsequence $\{A_i\}_{2\le i\le\ell}$ has $f(x-i,\ell-1)$ possibilities. Hence \[f(x,\ell)=\sum_{i=0}^x\text C_x^if(x-i,\ell-1).\] By applying this formula and only considering modulo $10$, we get $f(1,2)=3$, $f(1,3)=4$, $f(1,4)=5$, $f(1,5)=6$, $f(1,6)=7$, $f(1,7)=8$, $f(1,8)=9$, $f(1,9)=0$, $f(1,10)=1$, $f(2,2)=9$, $f(2,3)=6$, $f(2,4)=5$, $f(2,5)=6$, $f(2,6)=9$, $f(2,7)=4$, $f(2,8)=1$, $f(2,9)=0$, $f(2,10)=1$, $f(3,2)=7$, $f(3,3)=4$, $f(3,4)=5$, $f(3,5)=6$, $f(3,6)=3$, $f(3,7)=2$, $f(3,8)=9$, $f(3,9)=0$, $f(3,10)=1$, $f(4,2)=1$, $f(4,3)=6$, $f(4,4)=5$, $f(4,5)=6$, $f(4,6)=1$, $f(4,7)=6$, $f(4,8)=1$, $f(4,9)=0$, $f(4,10)=1$, $f(5,2)=3$, $f(5,3)=4$, $f(5,4)=5$, $f(5,5)=6$, $f(5,6)=7$, $f(5,7)=8$, $f(5,8)=9$, $f(5,9)=0$, $f(5,10)=1$, $f(6,2)=9$, $f(6,3)=6$, $f(6,4)=5$, $f(6,5)=6$, $f(6,6)=9$, $f(6,7)=4$, $f(6,8)=1$, $f(6,9)=0$, $f(6,10)=1$, $f(7,2)=7$, $f(7,3)=4$, $f(7,4)=5$, $f(7,5)=6$, $f(7,6)=3$, $f(7,7)=2$, $f(7,8)=9$, $f(7,9)=0$, $f(7,10)=1$, $f(8,2)=1$, $f(8,3)=6$, $f(8,4)=5$, $f(8,5)=6$, $f(8,6)=1$, $f(8,7)=6$, $f(8,8)=1$, $f(8,9)=0$, $f(8,10)=1$, $f(9,2)=3$, $f(9,3)=4$, $f(9,4)=5$, $f(9,5)=6$, $f(9,6)=7$, $f(9,7)=8$, $f(9,8)=9$, $f(9,9)=0$, $f(9,10)=1$, $f(10,2)=9$, $f(10,3)=6$, $f(10,4)=5$, $f(10,5)=6$, $f(10,6)=9$, $f(10,7)=4$, $f(10,8)=1$, $f(10,9)=0$, $f(10,10)=1$.

Lastly, we get $K\equiv\textstyle\sum\limits_{i=1}^{10}f(10,i)\equiv\boxed{\textbf{(C) } 5}\pmod{10}$. ~Quantum-Phantom

Solution 3 (Cheese, but this time it actually works)

Seeing that all the answers are different modulus 5, and that 10 is divisible by 5, we cheese this problem.

Let $A_1, A_2, \cdots, A_n$ be one sequence satisfying the constraints of the problem. Let $b_1, b_2, \cdots, b_n, b_{n+1}$ be the sequence of nonnegative integers such that $A_k$ has $b_k$ elements for all $1\leq{}k\leq{}n$, and $b_{n+1}=10$. Note that we can generate the number of valid sequences of $A$ by first generating all sequences of $b$ such that $b_i\leq{}b_{i+1}$ for all $1\leq{}i<n$, then choosing the elements from $A_{k+1}$ that we keep in $A_k$, given the sequence of $b$ as the restraint for the number of elements. For each sequence $b_1, b_2, \cdots{}, b_{n+1}$, there are $\prod_{i=2}^{n+1}\binom{b_i}{b_{i-1}}$ corresponding sequences for $A$. Now, consider two cases - either all terms in $b$ are either 0, 5, or 10, or there is at least one term in $b$ that is neither 0, 5 nor 10. In the second case, consider the last term in $b$ that is not 10 or 5, say $b_m$. However, that implies $b_{m+1}=10$, and so the number of corresponding sequences of $A$ is $\binom{10}{b_m}\cdot{}$ something or $\binom{5}{b_m}\cdot$ something, which is always a multiple of $5$. Therefore, we only need to consider sequences of $b$ where each term is $0$ $5$ or $10$. If all terms in $b$ are 0 or 10, then for each $1\leq{}n\leq{}10$ there are $n+1$ sequences of $b$ (since there are $n+1$ places to turn from $0$ to $10$), for a total of $2+3+\cdots+11=65\equiv0$ (mod 5). If there exists at least one term $b_k=5$, then we use stars and bars to count the number of sequences of $b$, and each sequence of $b$ corresponds to $\binom{10}{5}$ sequences of $A$. For each $n$, we must have at least one term of $5$. After that, there are $n-1$ stars and $2$ bars (separating $0$ to $5$ and $5$ to $10$), so that is $\binom{n+1}{2}$ sequences of $b$. So the sum is $\binom{2}{2}+\binom{3}{2}+\cdots+\binom{11}{2}=\binom{12}{3}\equiv0$ (mod 5). Therefore, the answer is 0 mod 5, and it must be $\boxed{\textbf{(C) } 5}$

Solution

We observe that in each sequence, if element $e \in A_i$, then $e \in A_j$ for all $j \geq i$. Therefore, to determine a sequence with a fixed length $n$, we only need to determine the first set $A_i$ that each element in $\left\{ 1, 2, \cdots , 10 \right\}$ is inserted into, or an element is never inserted into any subset.

We have \begin{align*} K & = \sum_{n = 1}^{10} \left( n + 1 \right)^{10} \\ & = \sum_{m = 2}^{11} m^{10} . \end{align*}

Recalling or noticing that $x^n \equiv x^{n \mod 4} \pmod {10}$, then, Modulo 10, we have \begin{align*} K & \equiv \sum_{m = 2}^{11} m^2 \\ & \equiv \sum_{m = 1}^{11} m^2 - 1^2 \\ & \equiv \frac{11 \cdot \left( 11 + 1 \right) \left( 2 \cdot 11 + 1 \right)}{6} - 1\\ & \equiv 505 \\ & \equiv \boxed{\textbf{(C) 5}}  . \end{align*}

~Steven Chen (Professor Chen Education Palace, www.professorchenedu.com)

Slightly easier, observe that $11^{10} \equiv 1^{10 }\pmod {10}$, so, working ${\mod 10}$, we have

\begin{align*} K & \equiv \sum_{m = 2}^{11} m^2 \\ & \equiv \sum_{m = 1}^{10} m^2 \\ & \equiv \frac{10 \cdot \left( 10 + 1 \right) \left( 2 \cdot 10 + 1 \right)}{6} \\ & \equiv \frac{10 \cdot 11 \cdot 21}{6} \\ & \equiv 5 \cdot 11 \cdot 7 \\ & \equiv \boxed{ 5} \end{align*}

~oinava

Video Solution 1 by OmegaLearn

https://youtu.be/0LLQW0XCKsQ

Video Solution

https://youtu.be/FpagLq1uzBI

~Steven Chen (Professor Chen Education Palace, www.professorchenedu.com)


See also

2023 AMC 12A (ProblemsAnswer KeyResources)
Preceded by
Problem 23
Followed by
Problem 25
1 2 3 4 5 6 7 8 9 10 11 12 13 14 15 16 17 18 19 20 21 22 23 24 25
All AMC 12 Problems and Solutions

The problems on this page are copyrighted by the Mathematical Association of America's American Mathematics Competitions. AMC logo.png